Talk:2023 AMC 12B Problems/Problem 15

Revision as of 06:59, 16 November 2023 by Jason 3007 (talk | contribs) (Is this answer wrong? 2023 amc12 b problem 15)
(diff) ← Older revision | Latest revision (diff) | Newer revision → (diff)

How can E be the right answer since when the gcb(210,c) is 1, gcb(a,14) and gcb(b,15) have to be 1. The relationship between them should be must, not or. Or do I made a mistake? (: